Autor Nachricht
Günther
BeitragVerfasst am: 25. Jul 2021 17:51    Titel:

TomS hat Folgendes geschrieben:

Du musst ein nicht-stationäres Szenario betrachten, also lokalisierte Wellenpakete, andernfalls hast du bereits per Konstruktion jedes Problem ausgeschlossen.

Ok, danke.
TomS
BeitragVerfasst am: 25. Jul 2021 16:31    Titel:

index_razor hat Folgendes geschrieben:
Ich glaube immer noch der ursprünglichen Frage liegt ein simpler Fehlschluß zugrunde: Ich habe eine Lösung mit Energie E1 und eine weitere Lösung mit Energie E2. Dann muß die Summe beider Lösungen die Energie E1 + E2 haben, ansonsten ist der Energieerhaltungssatz verletzt. Der Energieerhaltungssatz besagt aber nicht, daß die Energie der Summe gleich der Summe der Energien ist, sondern nur, daß die Energie der Summe (und jeder anderen Lösung) konstant ist.

Das ist wahrscheinlich ein Fehlschluss.

Der zweite könnte bei der Betrachtung stationärer Beispiele darin liegen, dass man meint, es verschwände etwas; tut es aber nicht, es war nie da, wie du richtig schreibst.
index_razor
BeitragVerfasst am: 25. Jul 2021 11:48    Titel:

ML hat Folgendes geschrieben:

Da der Knoten kein ausgedehntes Gebiet ist, ist sein Energieinhalt gleich null. Die Energie kann den Knoten insofern passieren, ohne jemals "drin" zu sein. Bei einem ausgedehnten Gebiet fehlt mir aber diese Anschauung. Da frage ich mich weiterhin, ob die Energie drumherum fließt oder durchgeht.


Energietransport findet genau dort statt, wo der Poynting-Vektor ungleich null ist. Also nicht in oder durch die Raumbereiche, in denen keine Felder vorhanden sind.

Mir ist nicht ganz klar welche Fragen noch offen sind. Die Frage, wohin bei destruktiver Interferenz die Energie verschwindet, geht anscheinend davon aus, daß es möglich ist, die Energie des Gesamtsystems zu verringern, anstatt lediglich die Energie einzelner Raumbereiche, ohne daß die Differenz irgendwo anders auftaucht. Es wird allerdings nicht klar formuliert wie, d.h. durch welche zeitlich veränderliche Feldkonfiguration das bewerkstelligt werden soll, sondern einfach vorausgesetzt.

Die Energie, die in realen Prozessen aus den Bereichen destruktiver Interferenz verschwindet, wandert in die Bereiche konstruktiver Interferenz (oder wird eventuell von Ladungen aufgenommen) und verschwindet nicht aus dem Gesamtsystem. Wenn von der Situation ausgegangen wird, daß sich zwei ebene Wellen oder irgendwelche anderen komplizierten Feldkonfigurationen komplett auslöschen, dann ist zu keinem Zeitpunkt irgendwo Energie gewesen, die hätte verschwinden können. Das resultierende Feld verschwindet einfach zu jedem Zeitpunkt.


P.S. Ich glaube immer noch der ursprünglichen Frage liegt ein simpler Fehlschluß zugrunde: Ich habe eine Lösung mit Energie E1 und eine weitere Lösung mit Energie E2. Dann muß die Summe beider Lösungen die Energie E1 + E2 haben, ansonsten ist der Energieerhaltungssatz verletzt. Der Energieerhaltungssatz besagt aber nicht, daß die Energie der Summe gleich der Summe der Energien ist, sondern nur, daß die Energie der Summe (und jeder anderen Lösung) konstant ist.
TomS
BeitragVerfasst am: 25. Jul 2021 11:11    Titel:

Das kannst du aber nicht mittels ebener Wellen verstehen, denn die sind unendlich ausgedehnt. Wenn in einem Bereich destruktive Interferenz vorliegt, dann überall, und das stellt kein Problem dar.

Du musst ein nicht-stationäres Szenario betrachten, also lokalisierte Wellenpakete, andernfalls hast du bereits per Konstruktion jedes Problem ausgeschlossen.
Günther
BeitragVerfasst am: 25. Jul 2021 10:39    Titel:

Anderes Szenario:

Wir erzeugen 2 ebene parallele Wasserwellen (2 parallele Bretter tauchen mit der entsprechenden Frequenz ein) so, dass die Wellen sich um pi phasenverschoben treffen.
Dann sollten sich, wenn ich es richtig sehe kinetische und potentielle Energie der beiden Wellen gerade "wegheben".

Mit anderen Worten, phasenrichtig addiert, ist die Energie der beiden Wellen Null und die Energieerhaltung gilt.

Wäre eine analoge Argumentation nicht auch mit elektromagnetischen Wellen möglich?
ML
BeitragVerfasst am: 25. Jul 2021 09:52    Titel:

Hallo,

TomS hat Folgendes geschrieben:
Im Falle einer stehenden Welle läuft die Energie eben gerade nicht über die Knoten hinweg; du hast eine stationäre Situation.

Ja, genau. Aber bringt uns das nicht ein wenig näher zu der Frage, was bei totaler Auslöschung von Wellen passiert? Wenn ich mir die stehende Welle als Überlagerung von zwei einzelnen, in entgegengesetzte Richtungen laufende Wellen, vorstelle, dann wäre der Energietransport über die Knoten ja trotzdem vorhanden.

Im Bild der Überlagerung würde man über die Knoten sagen, dass von links nach rechts die gleiche Leistung transportiert wird wie von rechts nach links und daher netto kein Energietransport stattfindet.

Da der Knoten kein ausgedehntes Gebiet ist, ist sein Energieinhalt gleich null. Die Energie kann den Knoten insofern passieren, ohne jemals "drin" zu sein. Bei einem ausgedehnten Gebiet fehlt mir aber diese Anschauung. Da frage ich mich weiterhin, ob die Energie drumherum fließt oder durchgeht.


Viele Grüße
Michael
TomS
BeitragVerfasst am: 25. Jul 2021 00:31    Titel:

Im Falle einer stehenden Welle läuft die Energie eben gerade nicht über die Knoten hinweg; du hast eine stationäre Situation.

Wir sind ja von der Frage ausgegangen, „wo die Energie hingeht“. Wenn du dir ein stationäres Interferenzmuster anschaust, dann „geht die Energie von den Bereichen der destruktiven Interferenz eben nirgendwo hin“.
ML
BeitragVerfasst am: 24. Jul 2021 22:36    Titel:

Hallo,

vielleicht können wir die Überlegungen mit einem mechanischen Analogon beginnen. Wir betrachten eine schwingende Gitarrensaite (Bildquelle: Von Qef - Own work by uploader, based on design of bitmap image Image:Overtone.jpg, Gemeinfrei, https://commons.wikimedia.org/w/index.php?curid=4344186).
Die stehenden Wellen kann man als Überlagerung einer nach rechts und einer nach links laufenden Welle betrachten. In den Knoten ist die Auslenkung gleich null. Als naive Frage würde ich gerne stellen: Wie kommt die Energie über die Knoten hinweg, ohne dass im Bereich der Knoten Energie vorliegt? "Läuft" die Energie überhaupt darüber hinweg angesichts der Tatsache, dass die Energie der einzelnen Schwinger sich gar nicht ändert.

Im Bereich des Lichtes fällt mir als eine mögliche Anordnung das Laser-Doppler-Anemometer ein. Hier erzeugt man mit zwei Laserstrahlen ein Interferenzstreifensystem mit hellen und dunklen Bereichen in Ausbreitungsrichtung des Lichtes (Bildquelle: https://www.youtube.com/watch?v=WL-7-47zWCY).

Beide Strahlen sind in guter Näherung Gaußstrahlen mit näherungsweise parallelem Licht.

Jetzt betrachten wir einen der dunklen Streifen. Fließt die Energie dort drum herum oder durch? Sind die Zusammenhänge vergleichbar mit der Gitarrensaite?


Viele Grüße
Michael
TomS
BeitragVerfasst am: 24. Jul 2021 17:34    Titel:

Ja, habe ich auch darüber nachgedacht; die trivialen Beispiele entsprechen sicher nicht unserem Verständnis von Interferenz.

Mein Vorschlag wäre,
- zwei räumlich lokalisierte Quellen
- davon ausgehend zwei Signale
- ein Bereich B innerhalb des gemeinsamen Vorwärtslichtkegels
- jedes der beiden Signale verschwindet nicht innerhalb von B
- die Superposition beider Signale verschwindet innerhalb von B

(die Dimension von B lasse ich bewusst offen)
[/list][/list]
index_razor
BeitragVerfasst am: 24. Jul 2021 16:49    Titel:

ML hat Folgendes geschrieben:

Aber die Frage ist andersherum auch spannend: Gibt es im Sinne der Maxwellgleichungen in einem endlichen Volumen eine komplette (nicht nur näherungsweise) Auslöschung zweier Lichtwellen, oder kann die komplette Auslöschung tatsächlich nur beispielsweise punktweise oder im Bereich einer endlichen Fläche auftreten?


Ich finde die Frage gar nicht so leicht zu präzisieren. Angenommen wir haben zwei Lösungen , die wir überlagern . Was charakterisiert nun Bereiche destruktiver Interferenz von E,B? Sicher gibt es viele Beispiele mit E=B=0 in ausgedehnten Raumbereichen, nämlich überall dort, wo die Signalfronten noch nicht eingetroffen sind. Das allein ist sicher nicht ausreichend um von "destruktiver Interferenz" zu reden. Wie kann man überhaupt destruktive Interferenz für beliebige Signalformen definieren?
index_razor
BeitragVerfasst am: 24. Jul 2021 16:45    Titel:

ML hat Folgendes geschrieben:

Meines Erachtens ist dieses Paradoxon jedoch ein ganz typischer Vertreter der Art "Wenn ich von einem physikalisch nicht realisierbaren Ausgangszustand ausgehe, brauche ich mich nicht zu wundern, wenn es nach logischen Folgerungen zu Widersprüchen kommt". Problematisch an der Frage ist nämlich letztlich schon die Annahme einer ebenen Welle mit unendlicher Energie und Leistung.


Problematisch ist doch allerdings schon die Annahme, daß man die Energien zweier interferierender Wellen einfach addieren dürfte. Das ist schon aus rein formalen Gründen falsch, unabhängig davon, ob solche Wellen physikalisch überhaupt realisierbar sind. Es ist eher so wie du weiter oben schreibst. Es verschwindet keine Energie, weil sie nie vorhanden war. (Bzw. wenn sie aus bestimmten Raumbereichen verschwindet, dann taucht sie in anderen Raumbereichen auf.)
TomS
BeitragVerfasst am: 24. Jul 2021 15:55    Titel:

Genau, letzteres ist die spannende Frage.

Man kann sicher eine Feldkonfiguration als Anfangsbedingung vorgeben, die innerhalb bestimmter Volumina exakt Null ist. Dann kann man in der Zeit vorwärts und rückwärts rechnen und erhält eindeutig eine frühere Anfangsbedingung, für die die Feldkonfiguration innerhalb kleinerer, eingeschlossener Volumina sogar über einen gewissen Zeitraum exakt verschwindet.

Alles das verletzt aber nicht den Energieerhaltungssatz.

Dieser wäre ohnehin nur verletzt, wenn sich über die Zeit etwas ändert, nicht jedoch, wenn sich zwei Feldkonfigurationen zur selben Zeit unterscheiden, also sicher nicht im stationären Fall.
ML
BeitragVerfasst am: 24. Jul 2021 15:35    Titel:

Hallo Tom,

TomS hat Folgendes geschrieben:
Warum ebene Wellen? Das ist zu restriktiv.
Willst du bestreiten, dass es im Allgemeinen tatsächlich Bereiche mit exakter Auslöschung gibt?


die Ausgangsfrage war:
"Es hat sich in meinem Physik LK die Frage aufgetan, wo die Energie paralellenen Lichtes bleibt bei einer totalen Auslöschung? Hat jemand eine Ahnung? Es gibt keine Maximas und keine Minimas. Nur eine komplette Auslöschung."

Hier ist von parallelem Licht die Rede, das sich überall auslöscht. Ich war hier von einer ebenen Welle ausgegangen, da meiner Vorstellung nach jede andere Konfiguration dazu führt, dass Energie seitlich (in Bezug auf die optische Achse bzw. die Ausbreitungsrichtung) transportiert wird und man dann nicht mehr von "parallelem Licht" sprechen kann.

Aber die Frage ist andersherum auch spannend: Gibt es im Sinne der Maxwellgleichungen in einem endlichen Volumen eine komplette (nicht nur näherungsweise) Auslöschung zweier Lichtwellen, oder kann die komplette Auslöschung tatsächlich nur beispielsweise punktweise oder im Bereich einer endlichen Fläche auftreten?

Viele Grüße
Michael
TomS
BeitragVerfasst am: 24. Jul 2021 15:28    Titel:

Warum ebene Wellen? Das ist zu restriktiv.

Willst du bestreiten, dass es im Allgemeinen tatsächlich Bereiche mit exakter Auslöschung gibt?
ML
BeitragVerfasst am: 24. Jul 2021 15:15    Titel:

Hallo,

TomS hat Folgendes geschrieben:
@Physik-Lehrer - ist die Frage für stationäre Situationen damit beantwortet?

ich hatte Dich nicht umsonst nach der genauen Konfiguration gefragt, für die Du das Problem siehst.

Ich frage mich nämlich, wie Du eine Auslöschung von E- und H-Feldern bei zwei ebenen Wellen hinbekommen willst, ohne dass diese Wellen zwangsläufig in die gleiche Richtung laufen und sich überall und zu jeder Zeit auslöschen.

Wenn Du das E- und H-Feld von Welle 1 durch ein E- und H-Feld einer Welle 2 dauerhaft auslöschen willst, so müssen E1 und E2 sowie H1 und H2 überall und zu jeder Zeit gegenphasig sein. Vergleicht man unter diesen Vorbedingungen die Poyntingvektoren der beiden Wellen, so zeigt dieser in beiden Fällen jeweils in die gleiche Richtung (Minus mal Minus ergibt Plus.).
Du hast dann eine Welle, die überall und zu jeder Zeit ein verschwindendes E-Feld und ein verschwindendes H-Feld hat. Die Frage, wo die "ausgelöschte" Energie hingegangen ist, stellt sich dann aber unter dem Vorzeichen: "War überhaupt Energie da?".

In diesem Zusammenhang kann man die Frage natürlich noch etwas allgemeiner fassen: "Wenn ich die Nullfunktionen für E- und H-Feld tatsächlich jeweils aus zwei sich gegenseitig auslöschenden Funktionspaaren E1/E2 und H1/H2 darstellen kann, wo kommt dann die Energie für die Funktionspaare her? Vorher war da doch nichts!?"

Meines Erachtens ist dieses Paradoxon jedoch ein ganz typischer Vertreter der Art "Wenn ich von einem physikalisch nicht realisierbaren Ausgangszustand ausgehe, brauche ich mich nicht zu wundern, wenn es nach logischen Folgerungen zu Widersprüchen kommt". Problematisch an der Frage ist nämlich letztlich schon die Annahme einer ebenen Welle mit unendlicher Energie und Leistung.


Viele Grüße
Michael
TomS
BeitragVerfasst am: 24. Jul 2021 12:49    Titel:

Für die Interferenz zweier Wellenpakete würde ich mit dem Poyntingschen Satz argumentieren.

Dieser besagt für





dass die zeitliche Änderung der Energie innerhalb eines Volumens V gerade dem Energiefluss durch die Oberfläche des Volumens entspricht:



Für zwei zunächst isolierte, gut lokalisierte Wellenpakete A und B, die durch die Oberfläche eintreten, innerhalb des Volumens interferieren und anschließend wieder isoliert austreten, gilt das Superpositionsprinzip: die Form des austretenden Wellenpaketes A ist unabhängig davon, ob Wellenpaket B vorliegt oder nicht (und umgekehrt). Jedes Wellenpaket wird nicht vom jeweils anderen Wellenpaket beeinflusst. Damit transportiert Wellenpaket A immer die selbe Energie in das Volumen V hinein und hinaus, unabhängig vom jeweils anderen Wellenpaket und der Interferenz beider irgendwo innerhalb des Volumens.

Also „Energie von A hinein ohne Existenz von B“ = „Energie von A hinaus ohne Existenz von B“ = „Energie von A hinein mit Existenz von B“ = „Energie von A hinaus mit Existenz von B“.

Der Poyntingsche Satz formalisiert diese Aussage für beliebige Feldkonfigurationen und Volumina.
TomS
BeitragVerfasst am: 24. Jul 2021 12:16    Titel:

@Physik-Lehrer - ist die Frage für stationäre Situationen damit beantwortet?
TomS
BeitragVerfasst am: 22. Jul 2021 17:19    Titel: Re: Unterricht

Physik-Lehrer hat Folgendes geschrieben:
Ich habe mir im Zuge meines Unterrichts heute die gleiche Frage gestellt und kam trotz längerer Suche noch zu keiner wirklich stichhaltigen (in irgend einer Publikation veröffentlichten) Erklärung.

Ich habe noch keine kurze, einfache Erklärung parat.

Im Falle eines üblichen Experimentes betrachtet man zumeist monochromatisches Licht und damit im wesentlichen eine stationäre (!) Situation. Die Energiedichte des elektromagnetischen Feldes ändert sich nicht mit der Zeit.

Bringt man nun eine zweite Lichtquelle (Spalt o.ä.) in den Aufbau des Experimentes ein, so ändert sich lediglich die Energiedichte des elektromagnetischen Feldes im Vergleich zum Setup ohne diese zweite Lichtquelle. Damit verschwindet aber nichts.

Man kann natürlich eine lokale Energie-Impuls-Erhaltungsgleichung formulieren, den sogenannten Poyntingschen Satz. Dessen universelle Gültigkeit folgt im wesentlichen aus den Maxwellschen Gleichungen. Allerdings sehe ich dazu aktuell keine anschauliche Erklärung.
Erster Admiral
BeitragVerfasst am: 22. Jul 2021 16:57    Titel:

Wenn ich das richtig verstehe ist der Aufbau so gedacht, dass zwei Lichtquellen in einem Abstand von ungeradzahligen vielfachen der halben Wellenlänge voneinander entfernt stehen und dabei ebene Wellen ausstrahlen die eine parallele oder antiparallel Laufrichtung aufweisen.

Das Problem ist, dass ebene Wellen eine komplette Idealisierung sind, die in der Realität so nicht vorkommen:
"Denn einerseits kann keine ebene Welle von einem endlich ausgedehnten Sender abgestrahlt werden und andererseits ist die Energie einer ebenen Welle unendlich." https://de.wikipedia.org/wiki/Ebene_Welle

sobald man nicht mehr von einer idealen ebenen Welle ausgeht ist das Problem gelöst, denn dann ist die Interferenz nur auf der direkten Verbindungslinie zwischen den Lichtquellen komplett destruktiv. Die Wellen breiten sich außenrum kreisförmig aus wodurch es außerhalb der direkten Verbindungslinie auch immer punkte gibt auf denen sie sich konstruktiv überlagern müssen
TomS
BeitragVerfasst am: 22. Jul 2021 08:59    Titel:

Ich komme heute Nachmittag dazu, was zu schreiben.
ML
BeitragVerfasst am: 22. Jul 2021 08:51    Titel: Re: Unterricht

Hallo,

Physik-Lehrer hat Folgendes geschrieben:
TomS hat Folgendes geschrieben:
Ist nach 15 Jahren nicht mehr sooo relevant, oder?


Ich habe mir im Zuge meines Unterrichts heute die gleiche Frage gestellt und kam trotz längerer Suche noch zu keiner wirklich stichhaltigen (in irgend einer Publikation veröffentlichten) Erklärung.


kannst Du die Frage konkreter formulieren? Oft besteht bei solchen Fragen das Problem, dass Konfigurationen angenommen werden, die in sich widersprüchlich sind bzw. physikalisch nicht vorkommen können. Daher braucht es ein konkreteres Set-up.

Viele Grüße
Michael
Physik-Lehrer
BeitragVerfasst am: 21. Jul 2021 22:33    Titel: Unterricht

TomS hat Folgendes geschrieben:
Ist nach 15 Jahren nicht mehr sooo relevant, oder?


Ich habe mir im Zuge meines Unterrichts heute die gleiche Frage gestellt und kam trotz längerer Suche noch zu keiner wirklich stichhaltigen (in irgend einer Publikation veröffentlichten) Erklärung.
TomS
BeitragVerfasst am: 26. Apr 2020 10:32    Titel:

Ist nach 15 Jahren nicht mehr sooo relevant, oder?
gaste4l4o40
BeitragVerfasst am: 26. Apr 2020 09:03    Titel: Re: Auslöschung von Energie ?

Prof hat Folgendes geschrieben:
Wenn zwei gegenphasige Fermionen (Materie und Antimaterie) zusammentreffen, entstehen Photonen. Wenn zwei gegenphasige Photonen aufeinandertreffen, entstehen bisher nicht bekannte "neutrale" Teilchen, die offenbar noch weniger wechselwirken als Neutrinos (Siehe auch die Erklärung des Energiedefizits, der zur Vorhersage von Neutrinos führte). Es wird also keine Energie vernichtet, sondern umgewandelt.


...das ist die beste Antwort soweit.
lordnaikon
BeitragVerfasst am: 25. Mai 2005 23:45    Titel:

auf zufälligem wege, habe ich diese seite hier gefunden!

http://www.ap.univie.ac.at/users/fe/MERLIN_MPI/konzept.htm

nun wollt eich euch daran teil haben lassen und habe mich zurückgemeldet

lustigerweise, hat es genau mein gedanken experiment als beispiel. die seite zeigte mir auf, das ich einen fehler in meinen überlegungen hatte. dieser wurde aber von auch nicht erkannt (vieleicht habt ihr mich auch einfach nicht verstanden) Augenzwinkern

aber thx für eure bemühungen Big Laugh
Nikolas
BeitragVerfasst am: 11. März 2005 20:56    Titel:

@ mr. Black:
Beim Mach-Zehner (ein halbdurchlässiger Spiegel in der Mitte) geht das Photon wieder zur Quelle, beim Michelson bin ich mir nicht ganz sicher.
doppelmuffe
BeitragVerfasst am: 11. März 2005 20:18    Titel:

hi,

ich hätte da mal ne frage: wie sind denn die strahlen in dem experiment aus dem anderen thread polarisiert? geschockt
mr. black
BeitragVerfasst am: 11. März 2005 20:16    Titel:

wo bleibt dann die Energie?
SheepTrick
BeitragVerfasst am: 11. März 2005 18:21    Titel:

ENIAC hat Folgendes geschrieben:

so... ich glaube, damit habt ihr's. die beiden lichtstrahlen, die sich (eindimensional) in derselben richtung ausbreiten, mit einer phasenverschiebung von pi/2, interferieren nicht, weil es halt 2 strahlen sind! die intensität über die gesamte weglänge ergibt sich einfach aus der summe der jeweiligen einzelintensitäten am entsprechenden ort.
damit ist dann auch der energieerhaltungssatz gerettet, analog wie beim doppelspalt.

cu all! :-)


Wie erklärst Du dann, dass man ein Michelson Interferometer genau so einstellen kann, dass im Detektor kein Photon mehr registriert wird ? Und wieso nennen die Physiker das dann "destruktive Interferenz" ?
Nikolas
BeitragVerfasst am: 11. März 2005 16:07    Titel:

Hier interferieren aber keine zwei seperate Strahlen sondern Wegmöglichkeiten. Nimm ein Michelson oder Mach-Zehnder-Interferometer und du hast exakt die gleiche Fragestellung und dein Argument ist vom Tisch.
ENIAC
BeitragVerfasst am: 11. März 2005 15:56    Titel:

wenn ich die diskussion bis jetzt richtig verfolgt habe, auch im parallel-thread, und eure argumentationen richtig verstanden, dann... bilde ich mir ein, dass euer gedankenexperiment ein gedankenexperiment bleiben muss, ergo: nicht experimentell verwirklicht werden kann, weil der versuchsaufbau falsch ist! (no offense!)

2 lichtstrahlen, die sich (eindimensional) in derselben richtung ausbreiten, und meinetwegen die richtige phasenverschiebung von pi/2 haben, löschen sich deshalb nicht über die gesamte weglänge aus, weil 2 lichtstrahlen nie miteinander interferieren!
interferieren tut immer nur ein photon mit sich selbst, und interferieren tut -- wie schon irgendwo oben richtig bemerkt -- die wahrscheinlichkeitsverteilung des photons. wenn man sich die beiden lichtstrahlen als jeweils ein einziges photon vorstellt, quasi als "einteilchen-lichtstrahlen", kommt man besser zur lösung des problems, wie ich meine.
analog liegen die verhältnisse nämlich beim doppelspalt-experiment, speziell dann, wenn man sich auch das mit einem einzelnen photon vorstellt: das photon existiert in einer superposition der beiden zustände "spalt 1" und "spalt 2", durchläuft damit quasi beide spalte gleichermassen, und wird dahinter, auf einem schirm oder so, wieder als ein einzelnes, ungeteiltes photon registriert. die beiden zustände, genauer gesagt, die wahrscheinlichkeitsverteilungen beider zustände, interferieren auf dem schirm, und resultieren in einer wahrscheinlichkeitsverteilung, die der (makroskopischen) intensitätsverteilung entspricht. wird das "einteilchen-experiment" hinreichend oft wiederholt, resultiert die (makroskopische) intensitätsverteilung als summe über alle registrierten photonen aller "einteilchen-experimente". orte, an denen sich, wie man sagt, destruktive interferenz ereignet, sind schlicht orte, an denen die wahrscheinlichkeitsverteilung null ist, also niemals ein photon registriert wird. es interferieren niemals 2 verschiedene photonen, sondern nur die wahrscheinlichkeitsverteilung eines jeden einzelnen photons mit sich selbst! -- ansonsten müssten sich ja an einem ort destruktiver interferenz 2 photonen auslöschen, also energie (bzw. intensität, leistung pro fläche) vernichtet werden. was allerdings "vernichtet" wird ist die wahrscheinlichkeit, irgendein photon an dieser stelle zu registrieren.

so... ich glaube, damit habt ihr's. die beiden lichtstrahlen, die sich (eindimensional) in derselben richtung ausbreiten, mit einer phasenverschiebung von pi/2, interferieren nicht, weil es halt 2 strahlen sind! die intensität über die gesamte weglänge ergibt sich einfach aus der summe der jeweiligen einzelintensitäten am entsprechenden ort.
damit ist dann auch der energieerhaltungssatz gerettet, analog wie beim doppelspalt.

cu all! :-)
Prof
BeitragVerfasst am: 25. Feb 2005 13:55    Titel: Auslöschung von Energie ?

Wenn zwei gegenphasige Fermionen (Materie und Antimaterie) zusammentreffen, entstehen Photonen. Wenn zwei gegenphasige Photonen aufeinandertreffen, entstehen bisher nicht bekannte "neutrale" Teilchen, die offenbar noch weniger wechselwirken als Neutrinos (Siehe auch die Erklärung des Energiedefizits, der zur Vorhersage von Neutrinos führte). Es wird also keine Energie vernichtet, sondern umgewandelt.
bishop
BeitragVerfasst am: 01. Feb 2005 20:11    Titel:

hmm meine Mathe lehrerin hat gemeint dass Frequenz = 0 nicht Energie = 0 bedeutet. D.h wir haben immer noch eine Welle und ein EM Feld drumherum nur ist die frequenz eben 0, sie wollte mir das anschaulich an einem simulationsprog zeigen, wir sind aber noch nicht dazu gekommen
jeanb
BeitragVerfasst am: 01. Feb 2005 19:58    Titel:

haben deine Doktoren irgendwas gewusst?!
Lehrling
BeitragVerfasst am: 23. Jan 2005 14:09    Titel:

Hi

Ich selber kann leider nicht zur Lösung des Problems beitragen, jedoch habe ich mehrere Doktoren der Physik auf meiner Schule, die können sich ja mal darüber den Kopf zerbrechen Augenzwinkern

Werde dann, sofern diese mir eine Auskunft geben können, diese hier reinschreiben.

Gruß,
Lehrling
bishop
BeitragVerfasst am: 22. Jan 2005 23:28    Titel:

tjo folks, das Problem greift um sich...
Ich hab mal darauf meine Physiklehrerin angesprochen, und sie bat um Bedenkzeit, weil sie es jetzt aus dem Stegreif auch nicht wusste, sie hat aber auch zu der Lösung tendiert, dass sich zwei Wellen niemals auf voller Länge auslöschen können, sondern eben nur auf einem bestimmten Abschnitt und die Energie wäre dann sozusagen woanders, aber sie meinte da steckt auch ein Fehler drin, mal sehen ob sie übers WE auf eine Lösung kam, ich werd euch da natürlich informieren^^
lordnaikon
BeitragVerfasst am: 22. Jan 2005 23:19    Titel:

es geht ja auch hauptsächlich nicht darum , das er 100% völlig weg ist .. sodern das z.b. 89 % oder so gut wie man es halt schaft , weg sind.

wenn nun aber die laser strahlen sich destruktiv interverieren , sind sie dann noch messbar? ich denke nicht

mal ne grundsätzliche frage:

bedeutet ein "nicht messebar" auch das es "weg" ist.

die einzel wellen werde ja bei interferenz nicht beeinflusst... wenn sich 2 destruktiv überlagern gibt es die teilwellen ja noch .. sie sind aber nicht messbar ... fordert das , das sie "weg" sind?
mr. black
BeitragVerfasst am: 22. Jan 2005 20:11    Titel:

Die Zweite Idee, die zwar leider nicht von mir ist ( Sie ist von Max Born ):

Das mit den Wellen stimmt doch gar nicht wirklich.
Jeanb, was du meinst ist die klassische Wellentheorie.

In wirklichkeit sind die Wellen doch nur Warscheinlichkeitswellen.
d.h. Die Photonen sind in wirklichkeit keine Wellen. Sie verhalten sich
nur so. Daher ist der Energieerhaltungssatz nicht verletzt, denn die
Photonen löschen sich nicht aus.
Die Warscheinlichkeiten, dass die Photonen einen Gewissen Ort
oder sonst eine Eigenschaft haben löschen sich aus.

Wenn du mal von der Seite in einen Laser schaust dann wirst du
hellere und dunklere stellen sehen.

Augenzwinkern
jeanb
BeitragVerfasst am: 22. Jan 2005 11:11    Titel:

also im Klartext:

Das Probleme ist irrelevant, da sich zwei Laser niemals vollständig auslöschen?

Dann muss ich mich wohl mit dieser Lösung anfreunden?

oder hat jemand noch eine Idee?
EXcimer
BeitragVerfasst am: 22. Jan 2005 10:33    Titel:

Das Problem bei dieser Frage ist meiner Ansicht nach folgendes:
Wenn man einen Laserstrahl betrachtet, dann hat dieser die unangenehme
Eigenschaft, dass er rechtwinklig zu seiner Ausbreitungrrichtung unendlich breit ist.
D.h. dass die transversale Energiedichtefunktion eines Laserstrahls zwar gegen Null konvergiert, aber eben niemals Null wird.
Damit wird es unmöglich, zu sagen, man bringt zwei Laserstrahlen zur Interferenz, weil immer irgendwo noch "Reste" beider Strahlen sind, die nicht interferieren.

Zur Vervollständigung:
Als transversale ( rechtwinklig zur Ausbreitungrsichtung ) Grenze eines Lasertrahls wird der Radius definiert, wo die Energiedichte gegenüber der Energiedichte im Zentrum ( maximale ED ) auf oder rd. 13,5% zurückgegangen ist.

Powered by phpBB © 2001, 2005 phpBB Group